what is 3.33333333333 as a whole number?

Answers

Answer 1

1/3 = 0.333333333333

Then

3.33333333 is a whole number

= 3 + 1/3


Related Questions

Translate to a system Reiko needs to mail her Christmas cards and packages and wants to keep her mailing costs to no more than $500. The number of cards is at least 4 more than twice the number of packages. The cost of mailing a card (with pictures enclosed) is $3 and for a package the cost is $7.

Answers

Given:

Let x be the number of the cards and y be the number of the package.

Given that the number of cards is at least 4 more than twice the number of packages.

[tex]x=2y+4[/tex]

Given that mailing costs no more than $500 and the cost of mailing a card is $3 and for a package, the cost is $7.

[tex]3x+7y=500[/tex]

Substitute x=2y+4 in this equation, we get

[tex]3(2y+4)+7y=500[/tex]

[tex]6y+8+7y=500[/tex]

[tex]13y=500-8[/tex]

[tex]y=\frac{492}{13}[/tex][tex]y=37.8[/tex]

Let y=37 and substitute in x=2y+4, we get

[tex]x=2\times37+4[/tex][tex]x=78[/tex]

Hence the number of cards = 78 and the number of packages =37.

The total cost for this is $493 not more than $500.

prove that 1+3+5+......2n-1=n²

Answers

As given by the question

There are given that the series

[tex]1+3+5+\cdots+(2n-1)=n^2[/tex]

Now,

For step 1:

Put n=1

Then LHS =1

And

[tex]\begin{gathered} R\mathrm{}H\mathrm{}S=(n)^2 \\ =(1)^2 \\ =1 \end{gathered}[/tex]

So,

[tex]\therefore L.H.S=R.H.S[/tex]

P(n) is true for n=1.

Now,

Step 2:

Assume that P(n) istrue for n=k

Then,

[tex]1+3+5+\cdots+(2n-1)=k^2[/tex]

Adding 2k+1 on both sides

So, we get:

[tex]1+3+5\ldots+(2k-1)+(2k+1)=k^2+(2k+1)=(k+1)^2[/tex]

P(n) is true for n=k+1

By the principle of mathematical induction P(n) is true for all natural numbers n.

Hence,

[tex]1+3+5+\cdots+(2n-1)=n^2[/tex]

For all n.

Hence proved.

h(x) =x² +9 if h(x)=9 , x =

Answers

The given expression as; h(x) =x² +9

for h(x) = 9

Substitute the value of h(x) = 9 in the given expression;

h(x) =x² +9

9 =x² +9

x² = 9 - 9

x² = 0

x = 0

Answer : x = 0

Using the hottest and coolest months data, find the equation for line of best fit for this data showing all steps by hand.

Answers

Let

x -----> average temperature

y ----> Electricity Bill

we take the points

(99,150) and (69,80)

step 1

Find out the slope

m=(80-150)/(69-99)

m=-70/-30

m=7/3

step 2

Find out the equation of the line in slope-intercept form

y=mx+b

we have

m=7/3

point (69,80)

substitute and solve for b

80=(7/3)(69)+b

b=80-161

b=-81

the equation is

y=(7/3)x-81

using a graphing tool

Remember that the value of y cannot be a negative number

Write each fraction in terms of the LCD.x2x + 12x - 1x + 13x22x – 111X + 1X + 13Need Help?Watch ItAdditional Materials

Answers

The given fractions are,

[tex]\frac{x^2}{2x-1},\text{ }\frac{x+1}{x+13}[/tex]

The LCD of fractions is the least common multiple of the denominators.

So, the LCD of the above fractions is,

[tex](2x-1)(x+13)[/tex]

Multiplying the numerator and the denominator of the fraction by a common term does not change the fraction.

So, the first fraction can be expressed in terms of the LCD as,

[tex]\frac{x^2}{2x-1}=\frac{x^2(x+13)}{(2x-1)(x+13)}[/tex]

The second fraction can be expressed in terms of the LCD as,

[tex]\frac{x+1_{}^{}}{x+13}=\frac{(x+1)(2x-1)}{(2x-1)(x+13)}[/tex]

Find the height of the cliff. If necessary, round to the nearest hundredth yard.

Answers

We are given a diagram showing a slope, and a vertical height. We now have what represents a right angled triangle. The distance from the base of the cliff to the end of the slope is given as 24 yards. The slope itself is 37 yards. We shall now determine the height of the cliff (from ground to top) as indicated.

Note that we shall use the Pythagoras' theorem which is;

[tex]c^2=a^2+b^2[/tex]

Where we have

[tex]\begin{gathered} c=\text{hypotenuse (longest side)} \\ a,b=\text{other sides} \end{gathered}[/tex]

We can now substitute the given values/side lengths and we'll have;

[tex]37^2=24^2+b^2[/tex][tex]1369=576+b^2[/tex]

Subtract 576 from both sides;

[tex]793=b^2[/tex]

Take the square root of both sides;

[tex]\begin{gathered} \sqrt[]{793}=\sqrt[]{b^2} \\ 28.160255\ldots=b \end{gathered}[/tex]

Rounded to the nearest hundredth, the answer now becomes;

ANSWER:

[tex]b=28.16yd[/tex]

The last option is the correct answer

(4.7 x 10-3) x 351Simplify the expressionusing scientific notation and express your answer(2.5 x 10') < (3.3 X 100)in scientific notation. Round your answer to the nearest thousandth.AnswerKeypadKeyboard Shortcutsx10

Answers

Given:

[tex]\frac{(4.7\times10^{-3})\times351}{(2.5\times10^5)\times(3.3\times10^6)}[/tex]

Remove the brackets and multiply common terms

[tex]\begin{gathered} \frac{(4.7\times10^{-3})\times351}{(2.5\times10^5)\times(3.3\times10^6)} \\ =\frac{4.7\times10^{-3}\times351}{2.5\times10^5\times3.3\times10^6} \\ =\frac{4.7\times351\times10^{-3}}{2.5\times3.3\times10^5\times10^6} \\ =\frac{1649.7\times10^{-3}}{8.25\times10^{11}} \end{gathered}[/tex]

Simplify further to get

[tex]\begin{gathered} \frac{1649.7\times10^{-3}}{8.25\times10^{11}} \\ =\frac{16497\times10^{-1}_{}\times10^{-3}}{825\times10^{-2}\times10^{11}} \\ =\frac{16497\times10^{-4}}{825\times10^9} \end{gathered}[/tex]

This further gives

[tex]\begin{gathered} \frac{16497\times10^{-4}}{825\times10^9} \\ =\frac{16497}{825}\times\frac{10^{-4}}{10^9} \\ =19.996\times10^{-4-9} \\ =19.996\times10^{-14} \end{gathered}[/tex]

Therefore, the answer is

[tex]19.996\times10^{-14}[/tex]

Given the function defined in the table below, find the average rate of change, in simplest form, of the function over the interval 2 ≤ x ≤ 6.

Answers

To find the average rate of change over an interval we need to calculate how much the function has changed over that interval by subtracting the final value by the initial one and dividing by the lenght of the interval. With this in mind we have:

[tex]\begin{gathered} \text{rate}=\frac{19-13}{6-2} \\ \text{rate}=\frac{6}{4} \\ \text{rate}=1.5 \end{gathered}[/tex]

The average rate of change for this interval is 1.5

A 51-inch TV suggests that the main diagonal of the TV is 51 inches. Determine the dimensions of the screen of a 51 -inch TV with a 16:9 aspect ratio.Please see attached photo

Answers

The aspect ratio 16:9 indicates the next relation between x and y:

[tex]\frac{y}{x}=\frac{16}{9}[/tex]

Applying the Pythagorean theorem to the right triangle formed:

[tex]51^2=x^2+y^2[/tex]

Isolating y from the first equation:

[tex]y=\frac{16}{9}x[/tex]

Substituting in the second equation:

[tex]\begin{gathered} 51^2=x^2+(\frac{16}{9}x)^2 \\ 2601=x^2+(\frac{16}{9})^2x^2 \\ 2601=x^2+\frac{16^2}{9^2}^{}x^2 \\ 2601=x^2+\frac{256}{81}^{}x^2 \\ 2601=\frac{337}{81}^{}x^2 \\ 2601\cdot\frac{81}{337}=x^2 \\ 625.166172=x^2 \\ \sqrt[]{625.17}\approx x \\ 25\approx x \end{gathered}[/tex]

Replacing in the equation of y:

[tex]\begin{gathered} y=\frac{16}{9}\cdot25 \\ y\approx44.44 \end{gathered}[/tex]

The approximate dimensions are:

length = 25 in

height = 44.44 in

Simplify.4n + 12 + 7n4 n + 1923 n16 n +711 n+ 12

Answers

11 n+ 12

In this expression, to simplify means to reduce it to the simpler expression. Hence:

1) Grouping similar terms

4n + 12 + 7n =

2) Adding them up:

4n+7n+12=

11n +12

An online company is advertising a mixer on sale for 35% off the original price of $224.99 what is the sale price for the mixer? Round your answer to the nearest cent, if necessary.

Answers

Given:

The original price of mixer is $224.99.

The discount on the mixer is 35%.

Explanation:

Determine the discount amount on the mixer.

[tex]\begin{gathered} d=\frac{35}{100}\cdot224.99 \\ =78.7465 \end{gathered}[/tex]

Determine the sale price of the mixer.

[tex]\begin{gathered} 224.99-78.7465=146.2435 \\ \approx146.24 \end{gathered}[/tex]

So sale price of the mixer is $146.24.

Given the vectors u =-7j and w=-9i+4j, find 8u and u+w.Write your answers in the form ai+bj.

Answers

Recall that:

[tex]\begin{gathered} \text{For all a, b, c, d, e real numbers:} \\ (ai+bj)+(ci+dj)=(a+c)i+(b+d)j, \\ e(ai+bj)=(ea)i+(eb)j\text{.} \end{gathered}[/tex]

Therefore:

[tex]\begin{gathered} 8u=8(-7j)=8(0i-7j)=(8\cdot0)i+(8\cdot(-7))j=0i-56j=-56j\text{.} \\ u+w=(-7j)+(-9i+4j)=(0i-7j)+(-9i+4j)=(0-9)i+(-7+4)j \\ =-9i-3j\text{.} \end{gathered}[/tex]

Answer:

[tex]\begin{gathered} 8u=-56j\text{.} \\ u+w=-9i-3j\text{.} \end{gathered}[/tex]

Given quadrilateral MNPQ which of the following set of conditons would not be enough to know that MNPQ is a parrelogram?

Answers

For a shape to be considered a parallelogram it has to meet the following conditions:

0. The opposite sides must be equal

,

1. The opposite sides are equal

,

2. Adjacent sides are supplementary

,

3. The diagonals bisect each other

,

4. The opposite sides are parallel

For the quadrilateral to be considered a parallelogram then, the conditions that should be met are:

MN=QP and MQ=NP

MN || QP and MQ || NP

The diagonals MP and NQ bisect each other.

∠M=∠P and ∠N=∠Q

From the given options, the second one and the third one are not enough to determine MNPQ as a parallelogram

find the volume round to the nearest tenth use 3.14 for pi 5km

Answers

Step 1

List all parameters

[tex]\begin{gathered} \pi\text{ = 3.14} \\ r\text{ = 5km} \\ \end{gathered}[/tex]

Step 2

Write the volume of a sphere

[tex]undefined[/tex]

the line contains the point (-3,5) and is perpendicular to the line y=3x-4

Answers

two lines are perpendicular when the multiplication of their slopes is equal to -1. The slope of y = 3x - 4 is 3. Then the slope of a perpendicular line ​is:

[tex]\begin{gathered} m\cdot3=-1 \\ m=-\frac{1}{3} \end{gathered}[/tex]

Slope-intercept form:

y = mx + b

where m is the slope and b is the y-intercept. Replacing with point (-3, 5) and m = -1/3, we get:

5 = -1/3(-3) + b

5 = 1+ b

5 - 1 = b

4 = b

Then, the equation is:

y = -1/3x + 4

on a map, the scale is 5 cm = 2km what is the missing distance?town A distance to 5.6km is the actual distance

Answers

Answer:

The distance on the map is 14 cm

Explanation:

Parameters:

Map scale: 5 cm = 2 km

Given actual distance = 5.6km

Let x be the distance on map, then

x = 5.6 km

2x = 5 * 5.6

2x = 28

x = 28/2

= 14 cm

The 7th grade took a field trip to the zoo. 50 students rode in cars and the rest of the students were split equally onto 4 buses. There are 142 total 7th graders. How many students were on each bus?

Answers

traveledGiven:

The total number of students is N = 142.

The number of students riding in a car is n(C) = 50.

The total number of buses is b = 4.

The objective is to find the number of students traveling on each bus.

Explanation:

Consider the number of students travelled in each bus as s.

Then, the total number of students traveling in 4 buses will be 4s.

The algebraic expression for the total number of students N can be represented as,

[tex]N=n(C)+b(s)\text{ . . . . .(1)}[/tex]

On plugging the given values in equation (1),

[tex]142=50+4s[/tex]

On further solving the above equation,

[tex]\begin{gathered} 142-50=4s \\ 4s=92 \\ s=\frac{92}{4} \\ s=23 \end{gathered}[/tex]

Hence, the number of students traveling on each bus is 23.

slove equations with variables on both sides-4k - 10 = -5k

Answers

We will investigate how to solve an equation consisting of one variable

We have the following equation at hand:

[tex]-4k\text{ -10 = -5k}[/tex]

The basic rule applied in solving equation like above is mathematical operations. We apply basic operations like:

[tex]\text{adding, subtracting, multiplying, division}[/tex]

on both sides of the equation accompained by a variable or a number in an attempt to isolate the variable ( k ).

To isolate the variable ( k ) we need all the terms involving the variable ( k ) on one side of the equation.

We will add ( 4k ) on both sides of the equation as follows:

[tex]\begin{gathered} -4k\text{ -10 + 4k= -5k + 4k} \\ (\text{ 4k - 4k ) - 10 = -k} \\ -10\text{ = -k} \end{gathered}[/tex]

Now to remove the negative sign accompained by ( k ) on the right hand side of the equation. We wil multiply both sides with ( -1 ) as follows:

[tex]\begin{gathered} -1\cdot(-10)\text{ = -1}\cdot(-k) \\ 10\text{ = k} \end{gathered}[/tex]

Hence, the value of ( k ) is:

[tex]10[/tex]

Write an expression for the operation described.

"5 divided by the product of 3 and 2"

A (5 ÷ 3) × 2(5 ÷ 3) × 2
B 3 × (2 ÷ 5)3 × (2 ÷ 5)
C (3 × 2) ÷ 5(3 × 2) ÷ 5
D 5 ÷ (3 × 2)

Answers

D]  5 ÷ (3 × 2) is the expression for the operation "5 divided by the product of 3 and 2".

The operation "5 divided by the product of 3 and 2" means that number 5 divided by the product of 3 and 2.

The mathematical representation of this operation is 5 ÷ (3 × 2).

The answer to this operation = 5 / 6.

Hence, 5 ÷ (3 × 2) is the expression for the operation "5 divided by the product of 3 and 2".

To understand more about multiplication and division refer -

https://brainly.com/question/28768606

#SPJ1

Question 1 of 14, Step 1 of 10/19CorrectDetermine if the following expression is a polynomial.4 – 8x + x²AnswerKeyboaO Yes O No

Answers

Solution

Given

[tex]4-8x+x^2[/tex]

We want to determine if it's a Polynomial

A polynomial is an expression consisting of indeterminates (also called variables) and coefficients, that involves only the operations of addition, subtraction, multiplication, and non-negative integer exponentiation of variables.

Hence 4 - 8x + x^2 is a Polynomial

The table shows the weights of bananas at a grocery store. Complete the table so that there is a proportional relationship between the number of bananas and their weight.Number Of Bananas. Weight In Kilograms. 2 ? 0.72 15 ?

Answers

Let u make the first box x and the second box y.

If there is a proportional relationship between the number of bananas and their weights, it means that:

[tex]\frac{2}{x}=\frac{6}{0.72}=\frac{15}{y}[/tex]

We can take the first pair and solve for x as follows:

[tex]\begin{gathered} \frac{2}{x}=\frac{6}{0.72} \\ 6x=2\times0.72=1.44 \\ x=\frac{1.44}{6} \\ x=0.24 \end{gathered}[/tex]

We can solve for y in the same manner:

[tex]\begin{gathered} \frac{6}{0.72}=\frac{15}{y} \\ 6y=15\times0.72=10.8 \\ y=\frac{10.8}{6} \\ y=1.8 \end{gathered}[/tex]

Therefore, the boxes are filled as shown below:

Answer this question

Answers

Okay, in this case the statement talks about the sum, according with this we need to find the sum of the number blue bikes (b) and 9 red bikes.

So, in this case the correct option is A. b+9 because it says sum

Given A = {(1, 3X-1, 5}(6, 4)), B = {(2, 0X4, EX-4, 5x0, 0)) and C = {(1, 1x0, 2x0, 3)(0, 4X-3, 5)), answer the following multiple
choice question:
From the list of sets A, B, and C above, choose the set of relations that correctly represents a function.
O Set A only
O Sets A and C only
O Sets A and B only

Answers

  The functions is Set A and Set B.

What is meant by  function?

A mathematical phrase, rule, or law that establishes the link between an independent variable and a dependent variable (the dependent variable). In mathematics, functions exist everywhere, and they are crucial for constructing physical links in the sciences.

A relation in a set is said to be a function, if every first element of an ordered pair in a set is  related with a unique element of a second element.

No, two distinct second elements of an ordered pair, has the same first element.

For, example, {(1,2), (1,3), (4,5)}, is not a function, but it is a relation.

In Ordered pair, (x, y)

x=First Element

y= Second Element

→In Set A

First Element              Second Element

1                                           3

-1                                           5

6                                          4

Every First  element of set A has a unique second element. So, it is a function.

→In Set B

First Element              Second Element

2                                          0

4                                          6

-4                                          5

0                                          0

Every First  element of set B has unique second element and no two distinct Second element of set B, has same first element. So, it is a function.

→In Set C

First Element              Second Element

1                                           1

0                                          2

0                                          3

-3                                          5

As, two same first elements of set C has distinct second element. So, it is not a function.

Therefore, Set A and Set B, are functions .

To learn more about function refer to:

https://brainly.com/question/25638609

#SPJ1

3. A toy box is 24 cm long, 15 cm wide and 11 cm high. What is the volume of the toy box? What is the correct number sentence for this problem? A.V=24×15×11B.V=24×15C.V=24×11D.V=15×11

Answers

ANSWER

[tex]\begin{gathered} V=24*15*11 \\ V=3960\text{ }cm^3 \end{gathered}[/tex]

EXPLANATION

The box is a rectangular prism. The volume of a rectangular prism is given by:

[tex]V=L*W*H[/tex]

where L = length

W = width

H = height

Therefore, the volume of the box can be written in the number sentence:

[tex]V=24*15*11[/tex]

and the volume of the box is:

[tex]V=3960\text{ }cm^3[/tex]

That is the answer.

the relationship between the minutes a candle is burned and the size of the candle in millimeters is shown on the graph.

Answers

The function is a decreasin line so the more time goes the side will decrease so the correct answer is:

The candle started at 9mm and shrinks 5mm every 4 minutes

What is the solution to the equation below? Round your answer to two decimal places.ln x = 0.2A.x = 1.58B.x = -0.70C.x = -1.61D.x = 1.22

Answers

Given the equation:

[tex]\ln \left(x\right)=0.2[/tex]

Apply the properties of logarithms:

[tex]e^{ln(x)}=e^{0.2}[/tex]

Simplify:

[tex]x=e^{0.2}=1.22[/tex]

Answer: D. x = 1.22

Find the cardinal number of the setWhere N denotes the set of all natural numbers

Answers

[tex]\begin{gathered} 3030 \\ x=\mleft\lbrace31,32,33,34\ldots\ldots\ldots\ldots\ldots\ldots\text{.}.55\mright\rbrace \\ x<56 \\ \end{gathered}[/tex]

And x is divisible by 6 . The natural number of x will be

[tex]x=\mleft\lbrace36,42,48,54\mright\rbrace[/tex]

Annie's backyard deck cost $61.75 per square meter to build. The deck is 7 meters wide and14 meters long. How much did it cost to build the deck?

Answers

ANSWER

the cost to build the deck is $6051.5

EXPLANATION

Given that;

The length of the deck is 14 m

The width of the deck is 7m

1 m^2 is equivalent to $61.75

Follow the steps below to find the cost to build the deck

Step 1; Find the area of the deck

[tex]\begin{gathered} \text{ Recall, that the deck is a rectangular shape} \\ \text{ Area of a rectangle = length }\times\text{ width} \\ \text{ Area of a reactangle = 14 }\times\text{ 7} \\ \text{ Area of a rectangle = 98m}^2 \end{gathered}[/tex]

Step 2; Find the total cost of the deck

Let x represents the total cost to build the deck

[tex]\begin{gathered} \text{ 1m}^2\text{ }\rightarrow\text{ \$61.75} \\ \text{ 98m}^2\text{ }\rightarrow\text{ \$x} \\ \text{ cross multiply} \\ \text{ 1m}^2\text{ }\times\text{ \$x = \$61.75 }\times\text{ 98m}^2 \\ \text{ Isolate \$x }\frac{}{} \\ \text{ \$x = }\frac{\text{ \$61.75}\times98\cancel{m^2}}{1\cancel{m^2}} \\ \text{ \$x = \$61.75 }\times\text{ 98} \\ \text{ \$x = \$6051.5} \end{gathered}[/tex]

Therefore, the cost to build the deck is $6051.5

what is 12/8 × 18/16

Answers

First of all, simplify the given fractions

The current student population of Kansas City is 2700. If the population increases at a rate of 5.2% each year. What will the student population be in 4 years?Write an exponential growth model for the future population P(x) where x is in years:p(x)=What will the population be in 4 years? (Round to nearest student)

Answers

ANSWER

P(x) = 2700(1.052)^t

P(4) = 3307. (Rounded to nearest student)

EXPLANATION

Given:

1. The current student population to be 2700

2. The growth rate = 5.2% = 0.052

Desired Outcome

1. The exponential growth model

2. Population of the students in 4 years

The Exponential Growth Model

[tex]\begin{gathered} P(x)\text{ = 2700\lparen1 + 0.052\rparen}^t \\ P(x)\text{ = 2700\lparen1.052\rparen}^t \end{gathered}[/tex]

Population in 4 years

[tex]\begin{gathered} P(4)\text{ = 2700\lparen1.052\rparen}^4 \\ P(4)\text{ = 2700}\times1.2248 \\ P(4)\text{ = 3306.96} \end{gathered}[/tex]

Hence, the Exponential Growth Model P(x) = 2700(1.052)^t and the Population of the students in 4 years P(4) = 3307. (Rounded to nearest student)

Other Questions
QUESTION IS IN IMAGE!! DONT SHOW WORK JUST THE ANSWER UNLESS YOU NEED TO Which of the functions below could possibly have created this graph?O A. F(x)=x+x+3O B. F(x)=1.9x +15x -6O C. F(x)=-x + 2x-3xO D. F(x) = 3x +7x +15x Im confused because I dont know how to answer this question What is the inverse equation for h(x) = 2log(x-3) ? Compared with other phases of the new-product development process, the largest number of new-product ideas is rejected during the ____ phase. 9+7x = -5Which is the following above Addition property of equality Subtraction property of equality Simplify Division property of equality Symmetric property of equality Distributive property Do capillaries do both Oxygen blood and reabsord nutrients ? And what does it mean it takes it from ICF and turns that into lymph (whats lymph and whats its purpose?) And where is the pancreas releasing the enzymes . Can you compare simple harmonic motion & hooks law. For me? If you randomly select a card from a well-shuffled standard deck of 52 cards, what is the probability that the card you select is a 7 or 5? (Your answer must be in the form of a reduced fraction.) In HIJ, the measure of J=90, the measure of I=62, and IJ = 86 feet. Find the length of HI to the nearest tenth of a foot. How might changes in phase relate to an issue in environmental racism? How do I graph y = -4x + 6 thanks! What value for x makes the following statement true? Select all the correct answersWhich two statements best describe the role of lymph in the human body?A It transports nutrients in the bodyB It produces hormones in the body.C It synthesizes antigens in the bodyD It circulates macrophages in the body.E It produces keratin in the body how does the concentration of prokaryotic cells at the surface of pelagic water compare with the concentrations at depths below 1000 m? f (x) = 2x -1find f (-4) 2.Lucy, a 12 year old 16 kg beagle needs a dose of the antibiotic cefazolin IV to treat her infected bite wound. The dose is 22 mg/kg IV. The concentration is 100 mg/ml. How many mgs of cefazolin will you give to Lucy? How many mls will you give to Lucy? find an equation for the line that passes through the points (-1,5) and (-5,1) A driver accelerates her car from rest to a velocity of 35.9 mi/hr in 8.2 seconds. What is the acceleration of the car? Santa has 3/4 of a bag of grain left. His hens eat 1/6 of a bag of grain every day. How many days will he be able to feed his hens?